Understanding Mean Value Theorem: Solving Homework Problems

Click For Summary
The discussion revolves around the Mean Value Theorem (MVT) and its application in solving homework problems. Participants clarify the distinctions between Rolle's Theorem and the MVT, noting that the latter is a generalization of the former. There is a focus on the continuity requirements of functions within the theorems, specifically that a function must be continuous on the closed interval [a, b] to apply these theorems correctly. Misunderstandings regarding the conditions for differentiability and continuity are addressed, emphasizing the importance of attention to detail in mathematical problems. Overall, the conversation highlights the nuances of applying theorems in calculus and the learning process involved.
Slimsta
Messages
189
Reaction score
0

Homework Statement


http://img14.imageshack.us/img14/6132/proiqc.jpg


Homework Equations





The Attempt at a Solution


the first 3 are from the textbook so they must be right.. the last 2 I am pretty sure i got right too..
because the 4th one, if f'(x)=0 then f(x)= c .. so its false.
im not too sure about the 5th one but I am 90% sure it should true.
 
Last edited by a moderator:
Physics news on Phys.org
(1) is Rolle's Theorem (Edit: this is incorrect - see the following posts)
(2) is the Mean Value Theorem, which is a generalization of Rolle's Theorem
I did not know (3) but that is interesting
(4) is correct, f(x) can be a constant function not equal to 0.
(5) is correct - take a(x) = f(x) - g(x). It's derivative is 0, so a(x) is constant (you can prove this using MVT).
 
Last edited:
VeeEight said:
(1) is Rolle's Theorem
(2) is the Mean Value Theorem, which is a generalization of Rolle's Theorem
I did not know (3) but that is interesting
(4) is correct, f(x) can be a constant function not equal to 0.
(5) is correct - take a(x) = f(x) - g(x). It's derivative is 0, so a(x) is constant (you can prove this using MVT).

i know that 1 and 2 are Rolle's Theorem and Mean Value Theorem, and its written exactly like in my textbook.. i can't find the mistake.

https://www.physicsforums.com/library.php?do=view_item&itemid=231 its even stated here..

so what is wrong?
 
Oh sorry, I missed that
(1) states that f is defined on [a,b] but it is not necessarily continuous there. It is continuous on (a,b) (since it is differentiable there) but not necessarily at a or b.
 
VeeEight said:
Oh sorry, I missed that
(1) states that f is defined on [a,b] but it is not necessarily continuous there. It is continuous on (a,b) (since it is differentiable there) but not necessarily at a or b.

thats a tricky question.. weird.
thanks a lot for your help man!
 
No problem, can't believe I missed that
In my experience, the physical science courses are where they try to 'trick' you like that, I've never had any questions like that when I was a math undergrad. I guess now you've learned that you have to pay attention to every little detail when writing tests in this class (which is probably a good idea, regardless). Cheers.
 
Question: A clock's minute hand has length 4 and its hour hand has length 3. What is the distance between the tips at the moment when it is increasing most rapidly?(Putnam Exam Question) Answer: Making assumption that both the hands moves at constant angular velocities, the answer is ## \sqrt{7} .## But don't you think this assumption is somewhat doubtful and wrong?

Similar threads

  • · Replies 11 ·
Replies
11
Views
2K
Replies
4
Views
2K
  • · Replies 1 ·
Replies
1
Views
2K
  • · Replies 2 ·
Replies
2
Views
2K
  • · Replies 3 ·
Replies
3
Views
1K
  • · Replies 1 ·
Replies
1
Views
2K
  • · Replies 14 ·
Replies
14
Views
2K
  • · Replies 3 ·
Replies
3
Views
1K
  • · Replies 6 ·
Replies
6
Views
2K
  • · Replies 12 ·
Replies
12
Views
3K